Convergence of $lim_{Ntoinfty}sum_{n=1}^N exp(-Nsin^2(frac{npi}{2N}))$ and $lim_{Ntoinfty}sum_{n=1}^N...











up vote
3
down vote

favorite
1












I can't find the right approach to tackle the question whether
$$lim_{Ntoinfty} sum_{n=1}^N expBigg(-N sin^2left(frac{npi}{2N}right)Bigg)$$
and
$$lim_{Ntoinfty} sum_{n=1}^N expBiggl(-sin^2left(frac{npi}{2N}right)Biggr)$$
converge or diverge. The fact that the limiting variable appears both as the upper bound of summation as well as in the individual summands seems to make the standard methods known to me inapplicable.



I suspect that the second limit (i.e. the one not containing $N$ in the exponent directly) does not exist, but that the first one may. I would be very grateful if you could point me to methods that allow one to determine the existence of the limits.



If a limit exists, I would also be very interested in understanding how, if at all, one could (approximately) replace the sum with an integral.



(For background, these questions have arisen during my study of the Rouse theory of polymer dynamics, e.g. in chapter 7.3.2. of Doi and Edwards, "The Theory of Polymer Dynamics". Physical explanations of how one can justify the treatment therein would be very welcome, too.)



Thank you in advance!










share|cite|improve this question
























  • ru.wikipedia.org/wiki/Логарифмический_признак_сходимости
    – Samvel Safaryan
    Nov 21 at 13:35






  • 1




    @SamvelSafaryan: Thanks for the link. I'm afraid, though, that this method is not applicable, or is it? I assume the theorem holds only if the $a_n$ do not depend on the limiting variable $N$ (i.e. the maximum index of summation). Please correct me if I'm wrong, I might just have conceptual difficulties.
    – Batista
    Nov 21 at 13:55










  • The second sum diverges to $infty $ because the same sum divided by $N$ tends to $int_{0}^{1}e^{-sin^2(pi x/2)},dx>0$.
    – Paramanand Singh
    Nov 21 at 14:21












  • @ParamanandSingh: That's an elegant argument, thanks!
    – Batista
    Nov 21 at 14:47






  • 2




    Replacing $e^{-Ncdots}$ with $e^{-N^2cdots}$ gives a more interesting problem with the limit converging to a finite value.
    – Winther
    Nov 21 at 23:40

















up vote
3
down vote

favorite
1












I can't find the right approach to tackle the question whether
$$lim_{Ntoinfty} sum_{n=1}^N expBigg(-N sin^2left(frac{npi}{2N}right)Bigg)$$
and
$$lim_{Ntoinfty} sum_{n=1}^N expBiggl(-sin^2left(frac{npi}{2N}right)Biggr)$$
converge or diverge. The fact that the limiting variable appears both as the upper bound of summation as well as in the individual summands seems to make the standard methods known to me inapplicable.



I suspect that the second limit (i.e. the one not containing $N$ in the exponent directly) does not exist, but that the first one may. I would be very grateful if you could point me to methods that allow one to determine the existence of the limits.



If a limit exists, I would also be very interested in understanding how, if at all, one could (approximately) replace the sum with an integral.



(For background, these questions have arisen during my study of the Rouse theory of polymer dynamics, e.g. in chapter 7.3.2. of Doi and Edwards, "The Theory of Polymer Dynamics". Physical explanations of how one can justify the treatment therein would be very welcome, too.)



Thank you in advance!










share|cite|improve this question
























  • ru.wikipedia.org/wiki/Логарифмический_признак_сходимости
    – Samvel Safaryan
    Nov 21 at 13:35






  • 1




    @SamvelSafaryan: Thanks for the link. I'm afraid, though, that this method is not applicable, or is it? I assume the theorem holds only if the $a_n$ do not depend on the limiting variable $N$ (i.e. the maximum index of summation). Please correct me if I'm wrong, I might just have conceptual difficulties.
    – Batista
    Nov 21 at 13:55










  • The second sum diverges to $infty $ because the same sum divided by $N$ tends to $int_{0}^{1}e^{-sin^2(pi x/2)},dx>0$.
    – Paramanand Singh
    Nov 21 at 14:21












  • @ParamanandSingh: That's an elegant argument, thanks!
    – Batista
    Nov 21 at 14:47






  • 2




    Replacing $e^{-Ncdots}$ with $e^{-N^2cdots}$ gives a more interesting problem with the limit converging to a finite value.
    – Winther
    Nov 21 at 23:40















up vote
3
down vote

favorite
1









up vote
3
down vote

favorite
1






1





I can't find the right approach to tackle the question whether
$$lim_{Ntoinfty} sum_{n=1}^N expBigg(-N sin^2left(frac{npi}{2N}right)Bigg)$$
and
$$lim_{Ntoinfty} sum_{n=1}^N expBiggl(-sin^2left(frac{npi}{2N}right)Biggr)$$
converge or diverge. The fact that the limiting variable appears both as the upper bound of summation as well as in the individual summands seems to make the standard methods known to me inapplicable.



I suspect that the second limit (i.e. the one not containing $N$ in the exponent directly) does not exist, but that the first one may. I would be very grateful if you could point me to methods that allow one to determine the existence of the limits.



If a limit exists, I would also be very interested in understanding how, if at all, one could (approximately) replace the sum with an integral.



(For background, these questions have arisen during my study of the Rouse theory of polymer dynamics, e.g. in chapter 7.3.2. of Doi and Edwards, "The Theory of Polymer Dynamics". Physical explanations of how one can justify the treatment therein would be very welcome, too.)



Thank you in advance!










share|cite|improve this question















I can't find the right approach to tackle the question whether
$$lim_{Ntoinfty} sum_{n=1}^N expBigg(-N sin^2left(frac{npi}{2N}right)Bigg)$$
and
$$lim_{Ntoinfty} sum_{n=1}^N expBiggl(-sin^2left(frac{npi}{2N}right)Biggr)$$
converge or diverge. The fact that the limiting variable appears both as the upper bound of summation as well as in the individual summands seems to make the standard methods known to me inapplicable.



I suspect that the second limit (i.e. the one not containing $N$ in the exponent directly) does not exist, but that the first one may. I would be very grateful if you could point me to methods that allow one to determine the existence of the limits.



If a limit exists, I would also be very interested in understanding how, if at all, one could (approximately) replace the sum with an integral.



(For background, these questions have arisen during my study of the Rouse theory of polymer dynamics, e.g. in chapter 7.3.2. of Doi and Edwards, "The Theory of Polymer Dynamics". Physical explanations of how one can justify the treatment therein would be very welcome, too.)



Thank you in advance!







sequences-and-series limits convergence summation exponential-sum






share|cite|improve this question















share|cite|improve this question













share|cite|improve this question




share|cite|improve this question








edited Nov 21 at 16:58

























asked Nov 21 at 12:29









Batista

185




185












  • ru.wikipedia.org/wiki/Логарифмический_признак_сходимости
    – Samvel Safaryan
    Nov 21 at 13:35






  • 1




    @SamvelSafaryan: Thanks for the link. I'm afraid, though, that this method is not applicable, or is it? I assume the theorem holds only if the $a_n$ do not depend on the limiting variable $N$ (i.e. the maximum index of summation). Please correct me if I'm wrong, I might just have conceptual difficulties.
    – Batista
    Nov 21 at 13:55










  • The second sum diverges to $infty $ because the same sum divided by $N$ tends to $int_{0}^{1}e^{-sin^2(pi x/2)},dx>0$.
    – Paramanand Singh
    Nov 21 at 14:21












  • @ParamanandSingh: That's an elegant argument, thanks!
    – Batista
    Nov 21 at 14:47






  • 2




    Replacing $e^{-Ncdots}$ with $e^{-N^2cdots}$ gives a more interesting problem with the limit converging to a finite value.
    – Winther
    Nov 21 at 23:40




















  • ru.wikipedia.org/wiki/Логарифмический_признак_сходимости
    – Samvel Safaryan
    Nov 21 at 13:35






  • 1




    @SamvelSafaryan: Thanks for the link. I'm afraid, though, that this method is not applicable, or is it? I assume the theorem holds only if the $a_n$ do not depend on the limiting variable $N$ (i.e. the maximum index of summation). Please correct me if I'm wrong, I might just have conceptual difficulties.
    – Batista
    Nov 21 at 13:55










  • The second sum diverges to $infty $ because the same sum divided by $N$ tends to $int_{0}^{1}e^{-sin^2(pi x/2)},dx>0$.
    – Paramanand Singh
    Nov 21 at 14:21












  • @ParamanandSingh: That's an elegant argument, thanks!
    – Batista
    Nov 21 at 14:47






  • 2




    Replacing $e^{-Ncdots}$ with $e^{-N^2cdots}$ gives a more interesting problem with the limit converging to a finite value.
    – Winther
    Nov 21 at 23:40


















ru.wikipedia.org/wiki/Логарифмический_признак_сходимости
– Samvel Safaryan
Nov 21 at 13:35




ru.wikipedia.org/wiki/Логарифмический_признак_сходимости
– Samvel Safaryan
Nov 21 at 13:35




1




1




@SamvelSafaryan: Thanks for the link. I'm afraid, though, that this method is not applicable, or is it? I assume the theorem holds only if the $a_n$ do not depend on the limiting variable $N$ (i.e. the maximum index of summation). Please correct me if I'm wrong, I might just have conceptual difficulties.
– Batista
Nov 21 at 13:55




@SamvelSafaryan: Thanks for the link. I'm afraid, though, that this method is not applicable, or is it? I assume the theorem holds only if the $a_n$ do not depend on the limiting variable $N$ (i.e. the maximum index of summation). Please correct me if I'm wrong, I might just have conceptual difficulties.
– Batista
Nov 21 at 13:55












The second sum diverges to $infty $ because the same sum divided by $N$ tends to $int_{0}^{1}e^{-sin^2(pi x/2)},dx>0$.
– Paramanand Singh
Nov 21 at 14:21






The second sum diverges to $infty $ because the same sum divided by $N$ tends to $int_{0}^{1}e^{-sin^2(pi x/2)},dx>0$.
– Paramanand Singh
Nov 21 at 14:21














@ParamanandSingh: That's an elegant argument, thanks!
– Batista
Nov 21 at 14:47




@ParamanandSingh: That's an elegant argument, thanks!
– Batista
Nov 21 at 14:47




2




2




Replacing $e^{-Ncdots}$ with $e^{-N^2cdots}$ gives a more interesting problem with the limit converging to a finite value.
– Winther
Nov 21 at 23:40






Replacing $e^{-Ncdots}$ with $e^{-N^2cdots}$ gives a more interesting problem with the limit converging to a finite value.
– Winther
Nov 21 at 23:40












2 Answers
2






active

oldest

votes

















up vote
3
down vote



accepted










Here's another way to see the limit of the first series is $infty.$ Verify that for each fixed $n,$



$$lim_{Nto infty}expleft(-N sin^2left(frac{npi}{2N}right)right) = 1.$$



Fix $N_0in mathbb N.$ Then for $Nge N_0,$ the first series is at least



$$sum_{n=1}^{N_0}expleft(-N sin^2left(frac{npi}{2N}right)right).$$



This is a finite sum, so computing its limit as $Nto infty$ is easy: we get $sum_{n=1}^{N_0}1= N_0.$ Since $N_0$ is arbitrarily large, the limit of the first series is $infty.$ Since the terms of the second series are at least as large as the those of the first series, the limit for the second series is also $infty.$






share|cite|improve this answer























  • Thank you very much, that's a very elegant solution!
    – Batista
    Nov 22 at 16:28


















up vote
3
down vote













Both sums diverge and a general idea to prove it is to use Riemann sums, just as suggested in the comments. The second sum was proved divergent in the comments. For the first sum notice that $0 le sin x le x$ for $x in [0, pi/2]$ implies $0 geq - sin^2 x geq -x^2$ in this range and since the exponential function is increasing we have
begin{align*}
sum_{n=1}^N expBigg(-N sin^2left(frac{npi}{2N}right)Bigg)
&geq sum_{n=1}^N expBigg(- frac{n^2pi^2}{4N}Bigg) \
&= sqrt{N} cdot left[ frac{1}{sqrt{N} } sum_{n=1}^N expBigg(- frac{pi^2}{4} bigg(frac{n}{sqrt{N}}bigg)^2 Bigg)right]
end{align*}

Now, the expression inside square brackets is close to the integral (you can compare them using integral test for instance):
$$
int_0^{sqrt{N}} e^{-frac{pi^2}{4} x^2} dx to int_0^infty e^{-frac{pi^2}{4} x^2} dx < infty quad text{as $N to infty$}.
$$

This implies the first integral grows at least as a multiple of $sqrt{N}$ and must diverge.






share|cite|improve this answer





















  • Thank you so much for your insightful answer! I wish I could accept both answers.
    – Batista
    Nov 22 at 16:22











Your Answer





StackExchange.ifUsing("editor", function () {
return StackExchange.using("mathjaxEditing", function () {
StackExchange.MarkdownEditor.creationCallbacks.add(function (editor, postfix) {
StackExchange.mathjaxEditing.prepareWmdForMathJax(editor, postfix, [["$", "$"], ["\\(","\\)"]]);
});
});
}, "mathjax-editing");

StackExchange.ready(function() {
var channelOptions = {
tags: "".split(" "),
id: "69"
};
initTagRenderer("".split(" "), "".split(" "), channelOptions);

StackExchange.using("externalEditor", function() {
// Have to fire editor after snippets, if snippets enabled
if (StackExchange.settings.snippets.snippetsEnabled) {
StackExchange.using("snippets", function() {
createEditor();
});
}
else {
createEditor();
}
});

function createEditor() {
StackExchange.prepareEditor({
heartbeatType: 'answer',
convertImagesToLinks: true,
noModals: true,
showLowRepImageUploadWarning: true,
reputationToPostImages: 10,
bindNavPrevention: true,
postfix: "",
imageUploader: {
brandingHtml: "Powered by u003ca class="icon-imgur-white" href="https://imgur.com/"u003eu003c/au003e",
contentPolicyHtml: "User contributions licensed under u003ca href="https://creativecommons.org/licenses/by-sa/3.0/"u003ecc by-sa 3.0 with attribution requiredu003c/au003e u003ca href="https://stackoverflow.com/legal/content-policy"u003e(content policy)u003c/au003e",
allowUrls: true
},
noCode: true, onDemand: true,
discardSelector: ".discard-answer"
,immediatelyShowMarkdownHelp:true
});


}
});














draft saved

draft discarded


















StackExchange.ready(
function () {
StackExchange.openid.initPostLogin('.new-post-login', 'https%3a%2f%2fmath.stackexchange.com%2fquestions%2f3007664%2fconvergence-of-lim-n-to-infty-sum-n-1n-exp-n-sin2-fracn-pi2n-a%23new-answer', 'question_page');
}
);

Post as a guest















Required, but never shown

























2 Answers
2






active

oldest

votes








2 Answers
2






active

oldest

votes









active

oldest

votes






active

oldest

votes








up vote
3
down vote



accepted










Here's another way to see the limit of the first series is $infty.$ Verify that for each fixed $n,$



$$lim_{Nto infty}expleft(-N sin^2left(frac{npi}{2N}right)right) = 1.$$



Fix $N_0in mathbb N.$ Then for $Nge N_0,$ the first series is at least



$$sum_{n=1}^{N_0}expleft(-N sin^2left(frac{npi}{2N}right)right).$$



This is a finite sum, so computing its limit as $Nto infty$ is easy: we get $sum_{n=1}^{N_0}1= N_0.$ Since $N_0$ is arbitrarily large, the limit of the first series is $infty.$ Since the terms of the second series are at least as large as the those of the first series, the limit for the second series is also $infty.$






share|cite|improve this answer























  • Thank you very much, that's a very elegant solution!
    – Batista
    Nov 22 at 16:28















up vote
3
down vote



accepted










Here's another way to see the limit of the first series is $infty.$ Verify that for each fixed $n,$



$$lim_{Nto infty}expleft(-N sin^2left(frac{npi}{2N}right)right) = 1.$$



Fix $N_0in mathbb N.$ Then for $Nge N_0,$ the first series is at least



$$sum_{n=1}^{N_0}expleft(-N sin^2left(frac{npi}{2N}right)right).$$



This is a finite sum, so computing its limit as $Nto infty$ is easy: we get $sum_{n=1}^{N_0}1= N_0.$ Since $N_0$ is arbitrarily large, the limit of the first series is $infty.$ Since the terms of the second series are at least as large as the those of the first series, the limit for the second series is also $infty.$






share|cite|improve this answer























  • Thank you very much, that's a very elegant solution!
    – Batista
    Nov 22 at 16:28













up vote
3
down vote



accepted







up vote
3
down vote



accepted






Here's another way to see the limit of the first series is $infty.$ Verify that for each fixed $n,$



$$lim_{Nto infty}expleft(-N sin^2left(frac{npi}{2N}right)right) = 1.$$



Fix $N_0in mathbb N.$ Then for $Nge N_0,$ the first series is at least



$$sum_{n=1}^{N_0}expleft(-N sin^2left(frac{npi}{2N}right)right).$$



This is a finite sum, so computing its limit as $Nto infty$ is easy: we get $sum_{n=1}^{N_0}1= N_0.$ Since $N_0$ is arbitrarily large, the limit of the first series is $infty.$ Since the terms of the second series are at least as large as the those of the first series, the limit for the second series is also $infty.$






share|cite|improve this answer














Here's another way to see the limit of the first series is $infty.$ Verify that for each fixed $n,$



$$lim_{Nto infty}expleft(-N sin^2left(frac{npi}{2N}right)right) = 1.$$



Fix $N_0in mathbb N.$ Then for $Nge N_0,$ the first series is at least



$$sum_{n=1}^{N_0}expleft(-N sin^2left(frac{npi}{2N}right)right).$$



This is a finite sum, so computing its limit as $Nto infty$ is easy: we get $sum_{n=1}^{N_0}1= N_0.$ Since $N_0$ is arbitrarily large, the limit of the first series is $infty.$ Since the terms of the second series are at least as large as the those of the first series, the limit for the second series is also $infty.$







share|cite|improve this answer














share|cite|improve this answer



share|cite|improve this answer








edited Nov 21 at 23:28

























answered Nov 21 at 21:02









zhw.

71.1k43075




71.1k43075












  • Thank you very much, that's a very elegant solution!
    – Batista
    Nov 22 at 16:28


















  • Thank you very much, that's a very elegant solution!
    – Batista
    Nov 22 at 16:28
















Thank you very much, that's a very elegant solution!
– Batista
Nov 22 at 16:28




Thank you very much, that's a very elegant solution!
– Batista
Nov 22 at 16:28










up vote
3
down vote













Both sums diverge and a general idea to prove it is to use Riemann sums, just as suggested in the comments. The second sum was proved divergent in the comments. For the first sum notice that $0 le sin x le x$ for $x in [0, pi/2]$ implies $0 geq - sin^2 x geq -x^2$ in this range and since the exponential function is increasing we have
begin{align*}
sum_{n=1}^N expBigg(-N sin^2left(frac{npi}{2N}right)Bigg)
&geq sum_{n=1}^N expBigg(- frac{n^2pi^2}{4N}Bigg) \
&= sqrt{N} cdot left[ frac{1}{sqrt{N} } sum_{n=1}^N expBigg(- frac{pi^2}{4} bigg(frac{n}{sqrt{N}}bigg)^2 Bigg)right]
end{align*}

Now, the expression inside square brackets is close to the integral (you can compare them using integral test for instance):
$$
int_0^{sqrt{N}} e^{-frac{pi^2}{4} x^2} dx to int_0^infty e^{-frac{pi^2}{4} x^2} dx < infty quad text{as $N to infty$}.
$$

This implies the first integral grows at least as a multiple of $sqrt{N}$ and must diverge.






share|cite|improve this answer





















  • Thank you so much for your insightful answer! I wish I could accept both answers.
    – Batista
    Nov 22 at 16:22















up vote
3
down vote













Both sums diverge and a general idea to prove it is to use Riemann sums, just as suggested in the comments. The second sum was proved divergent in the comments. For the first sum notice that $0 le sin x le x$ for $x in [0, pi/2]$ implies $0 geq - sin^2 x geq -x^2$ in this range and since the exponential function is increasing we have
begin{align*}
sum_{n=1}^N expBigg(-N sin^2left(frac{npi}{2N}right)Bigg)
&geq sum_{n=1}^N expBigg(- frac{n^2pi^2}{4N}Bigg) \
&= sqrt{N} cdot left[ frac{1}{sqrt{N} } sum_{n=1}^N expBigg(- frac{pi^2}{4} bigg(frac{n}{sqrt{N}}bigg)^2 Bigg)right]
end{align*}

Now, the expression inside square brackets is close to the integral (you can compare them using integral test for instance):
$$
int_0^{sqrt{N}} e^{-frac{pi^2}{4} x^2} dx to int_0^infty e^{-frac{pi^2}{4} x^2} dx < infty quad text{as $N to infty$}.
$$

This implies the first integral grows at least as a multiple of $sqrt{N}$ and must diverge.






share|cite|improve this answer





















  • Thank you so much for your insightful answer! I wish I could accept both answers.
    – Batista
    Nov 22 at 16:22













up vote
3
down vote










up vote
3
down vote









Both sums diverge and a general idea to prove it is to use Riemann sums, just as suggested in the comments. The second sum was proved divergent in the comments. For the first sum notice that $0 le sin x le x$ for $x in [0, pi/2]$ implies $0 geq - sin^2 x geq -x^2$ in this range and since the exponential function is increasing we have
begin{align*}
sum_{n=1}^N expBigg(-N sin^2left(frac{npi}{2N}right)Bigg)
&geq sum_{n=1}^N expBigg(- frac{n^2pi^2}{4N}Bigg) \
&= sqrt{N} cdot left[ frac{1}{sqrt{N} } sum_{n=1}^N expBigg(- frac{pi^2}{4} bigg(frac{n}{sqrt{N}}bigg)^2 Bigg)right]
end{align*}

Now, the expression inside square brackets is close to the integral (you can compare them using integral test for instance):
$$
int_0^{sqrt{N}} e^{-frac{pi^2}{4} x^2} dx to int_0^infty e^{-frac{pi^2}{4} x^2} dx < infty quad text{as $N to infty$}.
$$

This implies the first integral grows at least as a multiple of $sqrt{N}$ and must diverge.






share|cite|improve this answer












Both sums diverge and a general idea to prove it is to use Riemann sums, just as suggested in the comments. The second sum was proved divergent in the comments. For the first sum notice that $0 le sin x le x$ for $x in [0, pi/2]$ implies $0 geq - sin^2 x geq -x^2$ in this range and since the exponential function is increasing we have
begin{align*}
sum_{n=1}^N expBigg(-N sin^2left(frac{npi}{2N}right)Bigg)
&geq sum_{n=1}^N expBigg(- frac{n^2pi^2}{4N}Bigg) \
&= sqrt{N} cdot left[ frac{1}{sqrt{N} } sum_{n=1}^N expBigg(- frac{pi^2}{4} bigg(frac{n}{sqrt{N}}bigg)^2 Bigg)right]
end{align*}

Now, the expression inside square brackets is close to the integral (you can compare them using integral test for instance):
$$
int_0^{sqrt{N}} e^{-frac{pi^2}{4} x^2} dx to int_0^infty e^{-frac{pi^2}{4} x^2} dx < infty quad text{as $N to infty$}.
$$

This implies the first integral grows at least as a multiple of $sqrt{N}$ and must diverge.







share|cite|improve this answer












share|cite|improve this answer



share|cite|improve this answer










answered Nov 21 at 17:48









Daniel

1,516210




1,516210












  • Thank you so much for your insightful answer! I wish I could accept both answers.
    – Batista
    Nov 22 at 16:22


















  • Thank you so much for your insightful answer! I wish I could accept both answers.
    – Batista
    Nov 22 at 16:22
















Thank you so much for your insightful answer! I wish I could accept both answers.
– Batista
Nov 22 at 16:22




Thank you so much for your insightful answer! I wish I could accept both answers.
– Batista
Nov 22 at 16:22


















draft saved

draft discarded




















































Thanks for contributing an answer to Mathematics Stack Exchange!


  • Please be sure to answer the question. Provide details and share your research!

But avoid



  • Asking for help, clarification, or responding to other answers.

  • Making statements based on opinion; back them up with references or personal experience.


Use MathJax to format equations. MathJax reference.


To learn more, see our tips on writing great answers.





Some of your past answers have not been well-received, and you're in danger of being blocked from answering.


Please pay close attention to the following guidance:


  • Please be sure to answer the question. Provide details and share your research!

But avoid



  • Asking for help, clarification, or responding to other answers.

  • Making statements based on opinion; back them up with references or personal experience.


To learn more, see our tips on writing great answers.




draft saved


draft discarded














StackExchange.ready(
function () {
StackExchange.openid.initPostLogin('.new-post-login', 'https%3a%2f%2fmath.stackexchange.com%2fquestions%2f3007664%2fconvergence-of-lim-n-to-infty-sum-n-1n-exp-n-sin2-fracn-pi2n-a%23new-answer', 'question_page');
}
);

Post as a guest















Required, but never shown





















































Required, but never shown














Required, but never shown












Required, but never shown







Required, but never shown

































Required, but never shown














Required, but never shown












Required, but never shown







Required, but never shown







Popular posts from this blog

Quarter-circle Tiles

build a pushdown automaton that recognizes the reverse language of a given pushdown automaton?

Mont Emei